宏觀經濟學

內生增長:與 CRRA 實用程序的平衡增長路徑

  • April 13, 2016

由於溢出效應,我有一個內生增長模型。

$ \textbf{Model:} $

$$ K_t=\frac{1}{n}\sum_{t=1}^nk_t $$ 在這個模型中, $ k_t $ 由代理人選擇,並且 $ K_t=\bar{k}_t $ (所有的平均值 $ k_t $ ). 現在,代理人想要動態地最大化效用(在某些約束下)並且他們有 CRRA(恆定的相對風險厭惡)效用,所以最大化看起來像:

$$ \sum_{t=0}^\infty\beta^t\bigg(\frac{c_t^{1-\gamma}}{1-\gamma}\bigg) $$ $$ s.t.;Y_t=k_t^\alpha(E_tL)^{1-\alpha} $$ $$ c_t+i_t=Y_t $$ $$ k_{t+1}=(1-\delta)k_t+i_t $$ $$ c_t,i_t\geq0 $$ $ E_tL $ 是有效勞動,其餘變數是典型的(如果需要,我可以給出它們的定義)。

模型的最後一個補充是有兩個平衡約束:

$$ E_t=\frac{K_t}{L} $$ $$ k_t=K_t $$ $ \textbf{Solution:} $ 使用歐拉方程方法,目標中的兩項:

$$ …\frac{\beta^t[k_t^\alpha K_t^{1-\alpha}+(1-\delta)k_t-k_{t+1}]^{1-\gamma}}{1-\gamma}+\frac{\beta^{t+1}[k_{t+1}^\alpha K_{t+1}^{1-\alpha}+(1-\delta)k_{t+1}-k_{t+2}]^{1-\gamma}}{1-\gamma}… $$ 火: 寫 $ k_{t+1} $ 並替代消費:

$$ \beta^tc_t^{-\gamma}=\beta^{t+1}c_{t+1}^{-\gamma}[\alpha k_{t+1}^{\alpha-1}K_{t+1}^{1-\alpha}+1-\delta] $$ 代入平衡約束: $$ c_t^{-\gamma}=\beta c_{t+1}^{-\gamma}[\alpha+1-\delta] $$ $$ \implies \frac{c_{t+1}}{c_t}=[\beta(\alpha+1-\delta)]^{\frac{1}{\gamma}} $$ 這意味著消費以取決於偏好參數的恆定速率增長。 接下來我想證明我們有一條平衡的增長路徑。我的意思是所有變數都以相同的恆定速率增長。

$$ \frac{k_{t+1}}{k_t}=\frac{c_{t+1}}{c_t}? $$ 我已經開始回答這個問題,但我被卡住了。這是我到目前為止所擁有的: $$ k_{t+1}=k_t^\alpha K_t^{1-\alpha}+(1-\delta)k_t-c_t $$ 處於平衡狀態 $ K_t=k_t $ : $$ k_{t+1}=k_t+(1-\delta)k_t-c_t $$ $$ \implies \frac{k_{t+1}}{k_t}=1+(1-\delta)-\frac{c_t}{k_t} $$ 如果我們有一個恆定的資本增長率,假設: $$ \frac{k_{t+1}}{k_t}<\frac{c_{t+1}}{c_t} $$ 如果這是真的: $$ \underset{t\rightarrow \infty}{lim};\frac{k_{t+1}}{k_t}=\underset{t\rightarrow \infty}{lim};1+(1-\delta)-\frac{c_t}{k_t}=-\infty $$ 這意味著增長率將繼續下降。現在有兩種方法可以發生這種情況。第一種方法是如果 $ \underset{t\rightarrow \infty}{lim};k_t=-\infty $ ,這將清楚地表明情況並非如此,因為資本不可能為負數。第二種可能發生的方式是,如果 $ \underset{t\rightarrow \infty}{lim};k_t=D $ 在哪裡 $ D $ 是一些正的水平漸近線。如果它確實接近了某個漸近線,那麼就沒有理由不能證明消費的增長速度快於資本的增長速度。這就是我卡住的地方。我怎樣才能證明它不能收斂到某個漸近線,並且 $ \underset{t\rightarrow \infty}{lim};k_t=-\infty $ ? 另外,如果有更簡單的方法可以證明這個模型表現出平衡的增長路徑,它是什麼?任何幫助將不勝感激!

你已經獲得

$$ \frac{c_{t+1}}{c_t}=[\beta(\alpha+1-\delta)]^{\frac{1}{\gamma}} \equiv 1+g $$ 和

$$ \frac{k_{t+1}}{k_t}=1+(1-\delta)-\frac{c_t}{k_t} $$ 通過對等,您可以表明有一個獨特的規則可以保持平衡的增長路徑

$$ \frac{k_{t+1}}{k_t}=\frac{c_{t+1}}{c_t} \implies c_t =( 1-\delta-g)k_t $$ (順便說一句,消耗太多了)。這表明該模型具有獨特的均衡增長路徑。

如果你想進一步論證經濟確實會選擇這條路徑,你必須呼叫橫向條件(它限制了所選路徑對資本積累的影響),也許還有你選擇的效用函式滿足的 Inada 條件。

引用自:https://economics.stackexchange.com/questions/11531